Dalton CFP Sample Questions

Ace your homework & exams now with Quizwiz!

Jack is a knowledgeable and astute investor who is considering purchasing a permanent life insurance policy. He wants to achieve stock market-like returns for his cash value and pay a fixed premium. Which of the following insurance products is the most appropriate for him? A) Variable life. B) Universal life type A. C) Single premium whole life. D) Whole life.

A Among the types of life insurance listed, only variable life offers the ability to invest in subaccounts offering returns based on equity investments.

In March, Becky sold her XYZ stock to her brother Martin for $6,000 (its fair market value). Becky's adjusted basis in the stock was $8,200. In October, Martin sold the stock through his broker to an unrelated party for its fair market value of $7,200. Martin has told Becky that he will use her loss against his gain on the sale through his broker. Becky is concerned because she wanted to take the loss herself and is fearful of the IRS consequences if they both recognize the loss on their income tax returns. She has asked her planner to explain the tax treatment of these transactions on their tax returns. What does the planner tell Becky? Becky realized, Becky Recginized, Martin Realized, Martin Recognized: A) $2,200 loss$0 loss$1,200 gain$0 gain B) $0 loss$2,200 loss$0 gain$1,200 gain C) $0 loss$0 loss$0 gain$0 gain D) $2,200 loss$2,200 loss$1,200 gain$1,200 gain

A Because Becky sold her stock at a loss, she realized a $2,200 loss. Martin is a related party, thus, she may not recognize the loss for income tax purposes. When Martin sells the stock for a profit of $1,200, he will not recognize the gain because of the double-basis rule for sales of loss property to a related party. Sale of loss property to a related party results in a basis for gain of $8,200 and a basis for loss of $6,000. Because $7,200 is between the 2 basis amounts, Martin does not recognize the profit.

Jesse and Rhonda, ages 43 and 41 respectively, are meeting with their CFP® professional, Danielle. Today is January 3rd and the couple always insists on being her first client meeting of the year to review their prior year's financial performance. They have supplied Danielle with the following information: Repaid a $5,000 loan from Jesse's checking account held at a national bank. Their brokerage account increased from $75,000 to $80,000 due to capital appreciation. Purchased a $2,500 home theater system on the electronic store's credit card. Which of the following statements are CORRECT? The repayment of the loan using the checking account would increase their net worth by $5,000. Their net worth would increase by the same amount as the capital appreciation in their brokerage account. Jesse's checking account would be insured up to $250,000 by the FDIC. The couple's net worth is not affected by the purchase of the home theater system. A) 2, 3, and 4 B) 1, 2, and 4 C) 1, 3, and 4 D) 2 and 3

A Only statement 1 is not correct. The net effect on net worth using a checking account to pay off a debt is zero. The $5,000 increase in their brokerage account would cause their net worth to increase by the same amount. The addition of the home theater system increased their assets by $2,500, but simultaneously increased their liabilities but $2,500, resulting in a zero net effect on their net worth. Jesse's checking account in the national bank would be insured up to $250,000 by the FDIC.

Sara, age 51, has been working with Carol, a CFP® professional. Sara is the CEO of a large tax-exempt organization. Carol has developed a plan she feels will optimize Sara's retirement plan contributions and given it to her in a written document. Sara reviewed it and accepted all of the recommendations. The plan includes changing her contributions to the Section 403(b) plan and taking advantage of the available Section 457 plan. What else should Carol do to assist Sara in her retirement planning? Carol should discuss with Sara how monitoring the implemented recommendations will be accomplished and by whom. Carol should call the plan administrator for the plans at Sara's employer and get the forms for Sara to fill out. Carol should create a prioritized timeline for implementation of the recommendations. It is up to Sara to put Carol's recommendations in place, now that she has a good plan from a financial planning professional. A) 1 and 3 B) 3 only C) 1 and 2 D) 4 only

A Sara and Carol are in the implementation phase of the financial planning process. Carol should create a prioritized timeline for implementation of the recommendations and explain how monitoring the implemented recommendations will be accomplished and by whom. While Sara must contact the plan administrator herself, Carol may have told Sara that she should obtain the forms as part of her recommendations. The implementation of the recommendations may involve actions assigned to both Carol and Sara along with the timeline for implementation (Domain 6-implementing the Recommendations)

FP Board requires a written agreement by the CFP® certificant or the certificant's employer if a certificant provides financial planning or material elements of the financial planning process to a client. Which of the following (is)are required elements of this written agreement? The parties to the agreement The date and duration of the agreement How and on what terms each party can end the agreement Terms under which the certificant will use other entities to meet any of the agreement's obligations A) 1, 2, and 3 B) 1 and 4 C) 4 only D) 2 and 3

A Statements 1, 2, and 3 are correct. Statement 4 is incorrect. The terms under which the CFP® certificant will use other entities to meet any of the agreement's obligations may be communicated verbally or in writing. Another element that must be in writing addresses the services to be provided under the agreement. (Domain 8-Practicing Within Professional and Regulatory Standards)

Lance, a CFP® professional, has a long-time client, Angie, age 67. Angie is extremely wealthy and has no family to inherit her wealth. She does have many friends and charities she wishes to benefit during her lifetime and at death, and doing so is an established financial planning goal. Upon Lance's recommendation in previous years, Angie has established and funded various irrevocable trusts to benefit her favorite charities and some friends struggling with health issues. Angie does keep good records of the gifts she has made during the course of a year. She utilized all of her lifetime exemption amount and currently any noncharitable gift is taxable. As part of implementation of Lance's annual recommendations to Angie, he has assigned her with the task of notifying him if she makes a gift to anyone which totals more than the annual exclusion amount. Lance has also educated Angie about qualified gifts and urged her to use qualified gifts whenever possible. What else should Lance monitor regarding Angie's gifts to individuals? Lance should monitor the gifts for possible generation-skipping transfer tax issues. Lance should monitor any gift identified as a qualified transfer to be certain it meets the rules for a qualified transfer. Lance has nothing further to monitor once Angie was tasked with notifying Lance of gifts in excess of the annual exclusion. Lance should monitor the gifts to determine whether the gifts are of a present or future interest. A) 1, 2, and 4 B) 3 only C) 4 only D) 1 and 2

A Statements 1, 2, and 4 are correct. Lance should monitor Angie's gifts for all of these issues. (Domain 7-Monitoring the Recommendations)

Max and Annie have just had triplets. Max is a financial adviser and knows the importance of planning ahead and would like to begin saving for his children's education. Both Max and Annie expect their children to attend private colleges at the age of 18 for 4 years and expect tuition to be $20,000 per year in today's dollars per child. Annual inflation has averaged 4.5%, and tuition has been increasing at about 7% each year. Max and Annie are assuming they will earn an after-tax rate of return of 10% on their investments. How much should they save at the end of each year to pay for the college expenses of the triplets? Assume the last deposit is made when the children turn 18. Round your answer to the nearest $100. A) $17,100. B) $9,800. C) $11,250. D) $16,500.

A Step 1: Calculate the FV of the first year's tuition for 1 child and multiply the answer by 3. PV= −20,000n= 18i= 7PMT= 0FV= 67,598.65 × 3 (number of children) = 202,795.94 Step 2: Calculate the total amount due for all 3 children for the 4 years of college, remembering that tuition is paid at the beginning of the period. n= 4FV= 0i= [(1.10 ÷ 1.07) − 1] × 100 = 2.8037PMT= −202,795.94PVAD= 778,598.21 Step 3: Solve for the annual payments. n= 18PV= 0i= 10FV= 778,598.21PMT= -17,074.83, rounded to $17,100

Your client is going through the retirement plan selection process for his small firm with you. The client has asked questions about contribution costs, investment risk for employers and employees, flexibility of employer contributions to a plan, costs of administering a plan, and risks of achieving desired benefit amounts through a given retirement plan. In your discussions with the client, you need more information. What question should you ask next? A) What is the client's goal in establishing a retirement plan? B) Has the client brought an employee census so costs can be forecast for a selected plan? C) How much does the client want to pay for retirement plan benefits? D) Does the client want to establish a qualified plan or a tax-advantaged plan?

A Under Practice Standard 200-1, the financial planner and the client must mutually define and agree upon financial goals, needs, and priorities. In order to assist your client, you must first understand what goals and objectives he has for a retirement plan he will select. Is it to benefit all employees or a select few? Is it to attract, retain, or retire employees? Is it a tool to achieve tax benefits? Once the planner understands the client's objective in establishing a plan, then the process can move forward.

Lucy and Bob divorced 8 years ago, and Lucy was awarded custody of their daughter, Vicki. Under the divorce agreement, Lucy was to receive $95,000 the 1st year, $72,000 the 2nd year, and $45,000 annually for years 3 through last year. Beginning this year, when Vicki turns 18, the payments to Lucy will be reduced to $30,000 per year and will last until her death or remarriage. Lucy has come to her planner with some tax concerns on the alimony payment. She remembers the alimony recapture that was necessary several years ago and wants to know the impact for both herself and her ex-husband on this new reduction in alimony. What do you tell her? Lucy Bob A) $30,000 income30,000 deduction B) $45,000 income$45,000 deduction C) $5,500 income$5,500 deduction D) $0 income$0 deduction

A. Because the payments decrease when Vicki turns 18, amounts in excess of $30,000 are considered child support and the alimony paid for all relevant years is $30,000. Alimony is reported as income by the recipient and as a deduction by the payer.

All of the following statements regarding the treatment of incentive stock options and nonqualified stock options are correct EXCEPT: A) incentive stock options and nonqualified stock options will both create an AMT adjustment upon exercise. B) incentive stock options and nonqualified stock options can both be converted into stock and sold immediately. C) neither incentive stock options nor nonqualified stock options create taxable income at the date of grant. D) the exercise of incentive stock options will not result in a gain includable in W-2 income; however, exercising nonqualified stock options may result in W-2 income.

A. Nonqualified stock options will not create an AMT adjustment upon exercise; however, incentive stock options may create an AMT adjustment when exercised. All of the other statements are correct.

Delores, age 72, owns a chain of pastry shops, which is organized as a C corporation. She consults a CFP® professional for advice on transferring the business to her daughters, who are interested in taking over the business when she dies. After analyzing Delores's financial status and reviewing her goals, the CFP® professional determines that a transfer using a family limited partnership will best meet Delores's objectives. She presents the recommendation to Delores, who accepts it and asks the CFP® professional to begin implementing it. All of the following steps may take place in the implementation of this recommendation EXCEPT A) the daughters will assume control of the business once they collectively own a 51% limited partnership interest B) appraisals will be secured to support valuation discounts applied to the gifted partnership interests C) Delores will file gift tax returns claiming annual exclusions for the gifted partnership interests D) Delores will transfer the shares of stock in the business to a partnership

A. The daughters do not obtain any right to control the business by virtue of their ownership of the limited partnership interests they receive as gifts from Delores. Delores retains exclusive control over the business because she is the only partner who owns a general partnership interest. All of the other statements are correct. (Domain 6-Implementing the Recommendation(s))

You purchase one call contract and pay a $5 premium that allows you to buy the stock at $50. The stock is currently trading at $54. What is the intrinsic value of the situation you're in? A)-$4 B)$0 C)$2 D)$4

Answer D. Intrinsic Value of Call = Stock Price - Strike Price, therefore IV = 54 - 50 = 4.

Bob is a CFP® professional and has entered into a signed engagement letter to provide the first four steps of the financial planning process. Once Bob has made the recommendations, the engagement letter clearly identifies that the scope of the relationship ends and the client is solely responsible for implementation. Six months after Bob provided a client with his recommendations, the client approached Bob about purchasing life insurance through Bob, which was one of the recommendations in his plan. Is Bob still engaged with the client? A)No, the engagement letter limited the scope of the services to be provided. Bob is no longer engaged in the financial planning process with the client. Bob is now selling product beyond the scope of the engagement. B)Yes, Bob is still engaged with the client because Bob previously established a professional relationship and is now providing implementation services to the client by selling life insurance. C)No, a planner is not engaged with a client when only selling a single product. The planner would be engaged if applying multiple steps in the financial planning process. D)Yes, because even by providing one step in the financial planning process, a planner is considered engaged with a client.

Answer: B According to the CFP Board's Standard of Professional Conduct FAQs, in general, once a financial planning relationship with a CFP® professional has been established, all future services provided by the CFP® professional to the client are likely to be considered by CFP Board to be part of the financial planning process. Answer A is incorrect because CFP Board generally considers a CFP® professional to be engaged with the client for all future services provided by the CFP® professional. Answer C is incorrect because the planner is still engaged because a relationship was previously established and future services are considered to be part of the financial planning process. Answer D is incorrect because if the planner did not previously provide financial planning services and was solely selling life insurance, the planner would not be engaged with the client.

Cheryl is an insurance agent and a CFP® professional that works for a large, national insurance company. Recently, attorneys for the insurance company have required that any employees using the CFP® marks must remove the marks from their business cards, websites and promotional materials. The attorneys are concerned about potential lawsuits from customers alleging the CFP® professionals did not exercise the duty of a fiduciary. Based on this information, how should Cheryl proceed when working with clients? A)Cheryl must disclose in writing to her clients that although she is a CFP® professionals, her firm requires her to use the suitability standard and not the duty of a fiduciary. B)By complying with the attorney's request and removing the marks from all marketing materials, then Cheryl will no longer hold herself out as a CFP, she must only provide clients with a suitability duty of care. C)Cheryl should remove the CFP® marks as requested by the company attorneys, however she is still required to fulfill her professional obligations to follow the Standards of Professional Conduct and she still owes the duty of care of a fiduciary. D)As long as Cheryl removes the marks from all business cards, stationary, brochures, websites and does not hold her self out to the public as a CFP® professional, she is not professionally bound by the CFP Board's Standards of Professional Conduct.

Answer: C According to the CFP Board's Standard of Professional Conduct FAQs, removal of the CFP® marks from one's business cards or stationery does not relieve a CFP® professional of the obligation to follow the Standards of Professional Conduct. Answer A is incorrect because even though Cheryl does not hold herself out as a CFP® professional, she is still bound to the duties required by the Standards of Professional Conduct, including a fiduciary standard of care. Answer B is incorrect because Cheryl must still abide by the Standards of Professional Conduct, which require a fiduciary responsibility. Answer D is incorrect because the Standards of Professional Conduct still obligate Cheryl, even though she does not hold herself out as a CFP® professional.

Suppose you purchased 200 shares of XYZ stock on margin at $40 per share. Your initial margin was 50%, and you had a 35% maintenance margin. If XYZ drops to $25 per share, what amount will you be required to deposit? A) $3,000. B) $750. C) $0. D) $1,050.

B $40 × 50%=$20 (margin loan per share)$25 − $20 debt=$5 of equity$25 × 35%=$8.75 (required equity)$8.75 − $5=$3.75 per share required to deposit$3.75 × 200=$750

James asked Allison, his CFP® professional, for a review of his current investment portfolio because he is concerned about the tax implications of the Medicare tax on his investment income. Allison reviewed the investments, including real estate, and made recommendations to James. James has returned with the recommendations and told her there are changes he would like to make. What should Allison do? Allison should review the changes and determine the impact they would have on James investment income and exposure to the tax. After analysis, Allison should revise her recommendations and also present alternatives that align with her client's wishes. Allison should refer James to his CPA for detailed tax guidance on his investments. Allison should determine if the desired changes impact the overall financial plan. A) 1, 2, and 4 B) 1, 2, 3, and 4 C) 1 and 3 D) 3 only

B All of these statements are correct.

Lisa, a CFP® professional, has a client in the 35% marginal federal income tax bracket who would like to purchase a bond for her investment portfolio. The client is not subject to a state income tax. After making her calculations for the highest after-tax yield, which of the following bonds should Lisa recommend to her client? A) 3.75% private activity municipal bond B) 4.00% general obligation municipal bond C) 5.15% corporate debenture D) 2.55% U.S. Treasury note

B Lisa should recommend her client purchase the general obligation bond based on the following after-tax yield calculations (Domain 4-Developing the Recommendations): U.S. Treasury note [2.55% × (1 − .35)] 1.66%Corporate debenture [5.15% × (1 − .35)] 3.35%General obligation bond (tax-free municipal bond) 4.00%Private activity bond (tax-free municipal bond) 3.75%

Morgan is establishing a retirement plan for his small business. The cash flows for the business have been stable over the last 5 years and Morgan would like to invest some of this cash flow in a retirement plan to benefit him and his family member employees. As his planner, you have recommended several alternatives to be considered by Morgan. Which of the recommended plans are covered by Pension Benefit Guaranty Corporation insurance? Cash balance pension plan. Money purchase pension plan. Profit-sharing plan. SIMPLE. A) 1 and 2. B) 1 only. C) 1, 2, 3 and 4. D) 2 only.

B Pension Benefit Guaranty Corporation (PBGC) insurance only covers defined benefit and cash balance pension plans. Therefore, money purchase pension plans, profit-sharing plans, and SIMPLEs are not covered by PBGC insurance.

Which of the following statements regarding Medicare are CORRECT? Most Medicare beneficiaries do not pay a premium for Part A or Part B. Medicare covers individuals age 65 and over and individuals who are on kidney dialysis treatment. Medicare's Part A skilled nursing benefit covers claims as a result of Alzheimer's disease and dementia. Medicare Part B covers preventive services and drugs that cannot be self-administered. A) 2 and 3 B) 2 and 4 C) 1, 2, and 3. D) 1, 2, and 4.

B Statement 1 is incorrect; most Medicare beneficiaries do not pay a premium for Part A but enrollment in Part B requires the payment of a monthly premium. Statement 3 is also incorrect; Medicare's Part A skilled nursing benefit does not cover claims as a result of Alzheimer's disease and dementia since the patient's condition would not be expected to improve with treatment.

You have been conducting a review meeting with your client, Richard. He has indicated to you that he wishes to consider a purchase 500 shares of ABC common stock for his portfolio. However, he wants to make sure this purchase would be a prudent financial decision. Through your research, you discovered the stock paid a dividend of $1.50 this year. Also, this dividend is expected to grow at a constant rate of 2.5% for the next 3 years and at a rate of 4% thereafter. Richard has a required rate of return is 8% and the stock is current trading for $40.26 per share. Which of the following statements are CORRECT? ABC common stock is subject to default risk. ABC common stockholders have the right to vote for the board directors and to receive dividends (if declared). ABC common stock has an intrinsic value of $43.76. Richard should not consider buying ABC stock because the intrinsic value is lower than the market value. A) 2, 3, and 4 B) 2 and 4 C) 1 and 3 D) 1, 2, and 4

B Statement 1 is not correct, common stock is not subject to default risk. Statement 2 is correct, common stockholders have a number of rights as part owner of the corporation. Statement 3 is not correct, ABC common stock has a calculated intrinsic value of $37.01 (see calculation below). Statement 4 is correct, when a stock is selling in the market for greater than its intrinsic value, the stock would be considered overvalued and a purchase would not be warranted. 1. Compute the value of each future dividend until the growth rate stabilizes (Years 1-3). (Rounded to two decimal places) D1 = $1.50 × 1.025 = $1.54 D2 = $1.54 × 1.025 = $1.58 D3 = $1.58 × 1.025 = $1.62 2. Use the constant growth dividend discount model to compute the remaining intrinsic value of the stock at the beginning of the year when the dividend growth rate stabilizes (Year 4). D4 = $1.62 × 1.025 = $1.66 V = $1.66 ÷ (0.08 - 0.04) = $41.50 3. Use the uneven cash flow method to solve for the net present (intrinsic) value of the stock. CF0 = $0 CF1 = $1.54 CF2 = $1.58 CF3 = $1.62 + $41.50 = $43.12

Thomas was the designated beneficiary of his 72 year-old mother's qualified profit-sharing plan when she died earlier this year. The plan was valued at $1.5 million in his mother's gross estate. Thomas has elected not to receive a lump sum distribution from the plan. He also sold stock to a friend for $12,000. He had purchased the stock 2 years ago for $13,000 from his brother whose basis was $9,000. He contributed the proceeds from the stock sale to a Section 529 plan he established for his daughter. Which of the following statements you make to Thomas after your analysis is(are) CORRECT? Thomas will have an income tax deduction for some of the estate taxes paid on the profit-sharing plan by his mother's estate if he has taken distributions during the year. Thomas must receive a distribution before December 31 of this year because his mother died after her required beginning date; gifts to a Section 529 plan are always prorated over 5 years. Thomas is not required to receive a distribution from the profit-sharing plan until next year; he must include any distribution in his gross income for the year it is received. The related party rules do not apply to the stock sale; the cash he added to his daughter's Section 529 plan is eligible for the annual exclusion. A) 2 only B) 1, 3, and 4 C) 1 and 3 D) 3 and 4

B Statements 1, 3, and 4 are correct. Statement 2 is incorrect; Thomas is not required to begin distributions from the profit-sharing plan until the year after the year of his mother's death. Also, contributions to Section 529 plans are not always prorated over 5 years; a special rule allows a donor to contribute up to 5 times the gift tax annual exclusion amount in 1 year and have the contribution covered by the annual exclusion.

Which of the following statements is CORRECT regarding equity-based compensation? A) The exercise date for a nonqualified stock option (NQSO) creates W-2 compensation income subject to payroll taxes for the employee while providing a deduction for the employer. B) The grant date does not create a taxable event for incentive stock options (ISOs) but creates an AMT adjustment for nonqualified stock options (NQSOs). C) The exercise date for an incentive stock option (ISO) creates an AMT adjustment for the employer but does not create a taxable event for the employee. D) The shares received through the exercise of the NQSOs cannot be sold within two years from the date of the option's grant and one year from the date of the option's exercise, otherwise the favorable tax treatment will be lost.

B The grant date does not create a taxable event for an ISO or a NQSO. The exercise date for an incentive stock option (ISO) creates an AMT adjustment for the employee, not the employer. The shares received through the exercise of the ISOs cannot be sold within two years from the date of the option's grant and one year from the date of the option's exercise, otherwise the favorable tax treatment will be lost.

When your client, Carolyn, brought you her records you asked for to complete her income tax return for last year, you noted there were 3 receipts for charitable donations. Carolyn made the following contributions: A cash donation to the Red Cross of $400 Oil painting valued at $3,500 donated to the local art museum Sculpture valued at $7,000 donated to Feed-the-Homeless charity Carolyn expects to be able to deduct these contributions on her income tax return. You have determined her AGI is $21,000.The 3 charities are all public charities. What do you do next? A) You should calculate the maximum donation allowed on the painting and the sculpture for public charities. B) You should ask Carolyn for detailed records on the painting and the sculpture and how the FMV was determined. C) Because only basis is deductible, you should ask Carolyn for the receipts for the purchase of the painting and the sculpture. D) You should tell Carolyn she will have to carry forward $1,400 of her donation to a future year because her contributions exceed 50% of her AGI.

B To determine the basis of the painting and the sculpture, you need documentation on the purchase of the items. You also need to discover how FMV was determined. The substantiation letters from the charities will tell you what information they received from Carolyn but you need to validate both FMV and basis before determining the allowable income tax deduction.

Alberta, who is single and approaching retirement age, is a new client. She has received information from her retirement plan that she will receive a retirement benefit of $2,400 per month for the remainder of her life when she retires. She has been told she has a life expectancy of 18 years. She does know that the retirement plan is not a qualified plan. Alberta knows she made $30,000 in contributions to the plan and believes that her contributions will affect how much of her benefit will be taxable income. Alberta will also be receiving Social Security benefits of $2,100 monthly. She has engaged you, a financial planner, to explain to her how she should report her retirement income on her income tax return. What should you do next? A) Calculate the exclusion ratio for the monthly retirement benefit. B) Ask Alberta for information on all of the types of income she expects to receive during her retirement. C) Ask Alberta for all of the documentation she has on her retirement plan and the annuity benefits she is to receive from it. D) Tell Alberta her Social Security benefit is not includable in her taxable income.

B. Under Practice Standard 200-2, you will need to ask for more information and documentation. Before you can tell Alberta the effect her Social Security benefit and retirement annuity will have on her taxable income, you must know all of the types and amounts of any other income Alberta will receive. After analysis, these will determine how much of the Social Security benefit is includable in her gross income when added to the taxable portion of her annuity.

Homer is considering buying a custom-made RV in 10 years when he retires so that he and his wife can tour the country. If the RV currently costs $300,000 and inflation is 3%, how much money should he deposit at the beginning of each year to have enough to purchase the RV at the end of 10 years? Assume that Homer is earning a 12% rate of return on his investments. A) $23,499. B) $20,513. C) $19,475. D) $22,975.

B. PV = −300,000i = 3n = 10PMT = 0FV = 403,175 FV = 403,175PV = 0n = 10i = 12PMTAD = -20,513, or $20,513

Edgar invested $400,000 in ABC business last year, receiving a 30% nonpublicly traded limited partnership interest. Last year Edgar's share of the partnership's losses was $50,000, and this year his share of losses is $150,000. Last year Edgar reported $170,000 of income from other nonpublicly traded limited partnership interests he owns, and this year he will report $75,000 of nonpublicly traded limited partnership income. Which of the following statements is CORRECT? A) Edgar recognized $3,000 of losses last year and will take a $3,000 deduction this year. He is limited to $3,000 per year until ABC reports income with which to offset his remaining losses. B) Edgar could not take a deduction last year and will not take one this year, because ABC has not reported any income. Edgar can carry forward the $200,000 of losses to use against future income reported by ABC. C) Edgar deducted $50,000 of losses for last year but only $75,000 of losses for this year. He can carry forward the remaining $75,000 of losses to deduct against future income. D) Edgar deducted $50,000 of losses last year and will take a $150,000 deduction for losses this year.

C According to the passive-activity loss rules, Edgar may only deduct passive losses up to the amount of passive income earned in the year for nonpublicly traded limited partnership interests. The at-risk rules prohibit Edgar from taking losses in excess of his risk in the partnership, in this case, as defined by his original contribution. Under the passive-activity loss rules, Edgar may carry forward the $75,000 of suspended losses from this year to a year in which there is passive activity income that can be offset by the passive activity loss carried forward.

Carol, a CFP® professional, is having her initial meeting with a potential client, Fred. During the meeting, Carol learns that Fred works with her brother, Paul, and that Paul referred Fred to her. Paul told Fred that Carol was a great financial planner, but he did not tell him that Carol was his sister. What should Carol do next? A) Because no compensation is paid to Paul for the referral, Carol does not have to mention the relationship to her client. B) Carol should begin to establish what Fred's goals are for the engagement. C) Carol should disclose that Paul is her brother, even though there is no compensation paid to Paul for the referral. D) Carol should give Fred a list of documents that she will need to review in order to begin her analysis of his financial status.

C Even though there is no compensation to Paul for the referral, Carol should disclose the relationship to her client. Establishing Fred's goals and gathering documentation occurs later in the financial planning process. (Domain 2-Gathering Information Necessary to Fulfill the Engagement)

Heather, age 35, is a registered nurse at a local hospital. She is single and has a history of saving and investing since her early twenties. She has amassed a portfolio of $150,000 and is concerned that her money many not be positioned properly to meet her goals. She has engaged Lewis to help her develop a financial strategy. After a few meetings and analysis, he has prepared a recommendation of a managed account featuring a portfolio of 5 mutual funds. In what order should Lewis have completed the following steps? Have Heather sign his firm's letter of engagement. Provide Heather with the mutual fund prospectuses. Have Heather complete a risk-return profile questionnaire. Gather Heather's financial information, such as brokerage and bank statements. A) 1, 3, 4, 2 B) 2, 1, 4, 3 C) 4, 3, 1, 2 D) 3, 4, 1, 2

C First, Lewis should have gathered Heather's financial information during the initial interview. Next, Heather should have completed the risk-return profile questionnaire to determine the proper portfolio mix. Third, assuming they decided to work together, he should have had her sign his firm's letter of engagement before he arrived at the point of making recommendations. Lastly, Lewis should have presented her with the mutual fund prospectuses and any other necessary disclosures.

You have been provided with the following information regarding the past five year's returns on ABC mutual fund: Year 1, Return 12.5% Year 2, Return -15.3% Year 3 Return 8.7% Year 4 Return 3.4%Year 5, Return -1.3% Assuming this mutual fund has a normal distribution of returns, which of the following statements are CORRECT? The mean return for this series is 8.24%. The standard deviation for this series is 10.8% 50% of returns would be expected to be less than 1.6% 2.5% of returns would be expected to exceed 23.2% A) 1, 2, 3, and 4 B) 1, 2, and 4 C) 2, 3, and 4 D) 1 and 3

C Only statement 1 is not correct. For this series of historical returns, the arithmetic mean is 1.6% and the standard deviation is 10.8%. Fifty percent of returns lie above and below the mean of 1.6%. Only 2.5% of returns would be expected to exceed 23.2%, which is the remaining area to the right of two standard deviations to right of the mean. Ninety-five percent of returns fall within 2 standard deviations of the mean, but only 47.5% of these returns will exceed the mean leaving 2.5% to the right of 23.2%. Calculations: Arithmetic mean (12.5 - 15.3 + 8.7 + 3.4 - 1.3) ÷ 5 = 1.6% Standard deviation (keystrokes are shown for the HP 10BII+) [■] [C ALL] 12.5 [Σ+] 15.3 [+/-][Σ+] 8.7 [Σ+] 3.4 [Σ+] 1.3 [+/-][Σ+] [■] [Sx, Sy] 10.7991, or 10.8%

The Silvermans' asset allocation is as follows: U.S. Treasury securities30%Small-cap stocks10%Large-cap stocks40%S&P 500 ETF 20% Their portfolio is subject to which of the following risks? Market risk Default risk Portfolio manager risk Tax risk A) 1, 2, and 4. B) 2 and 3. C) 1, 3, and 4 D) 2, 3, and 4.

C The portfolio is not subject to default risk because they do not own any debt securities other than those issued by the U.S. government. Default risk is the risk that a business will be unable to service its debt obligations.

Margo and Jack are married and have come to you for advice about selecting a retirement plan for their business, Crackers Unlimited, Inc. The couple would like to establish a qualified retirement plan based on the following criteria: simplicity, tax deductible employer contributions, distribution of benefits in the form of cash, unrestricted ability to invest plan assets in company stock, in-service withdrawals allowed, and the ability to integrate with Social Security. What should you do next? A) You tell the clients that any plan that invests in the company stock will dilute their ownership share in the company. B) You should tell the Margo and Jack that nondiscrimination rules for qualified plans will limit the amount of contributions on their own behalf. C) You should ask your clients for a current census of its employees, including compensation and dates of employment with the company. D) After an analysis of the couple's requirements, you recommend a traditional profit-sharing plan as the best fit for Crackers Unlimited, Inc.

C Under Practice Standard 200-2, you should obtain sufficient quantitative information and documents relevant to the scope of the engagement. In order to perform any analysis and provide recommendations for suitable retirement plan alternatives, you must first know the number of employees, dates of hire, and compensation paid. With this information, you can perform the nondiscrimination tests for qualified plans and discover the effect the results may have on the types of plans that fulfill the couple's requirements. It is possible that a qualified plan may not be the best choice and you may need to recommend tax-advantaged or nonqualified plan alternatives when making your recommendations.

Which of the following statements are correct with regards to the taxation of insurance contract premiums and benefits? Group qualified long-term care insurance premiums are tax deductible for the employer while the benefits received are tax free up to a limit by the beneficiary. Group dental insurance premiums paid by the employer are tax deductible for the employer while the benefits are received tax-free by the employee. The death benefit from a modified endowment contract (MEC) is received tax free by the designated beneficiary assuming the contract is not subject to the transfer-for-value rule. Any payments from an annuity issued after December 31, 1986 that occur beyond the projected life expectancy of the owner/annuitant, are received as tax-free income by the annuitant. A) 1, 2, and 3 B) 1, 2, 3, and 4 C) 1 and 3 D) 1, 2, and 4

C or A. Not to clear but probably C Benefits paid from the group qualified LTC insurance plan to an individual are generally excludible from taxable income, subject to a $420 per day limitation in 2018. Any payments from an annuity issued after December 31, 1986, that occur beyond the projected life expectancy of the owner/annuitant, are fully taxable as ordinary income to the annuitant in the year received by the annuitant.

Snidely, a CFP® professional, met with Dudley and Geezer, Dudley's father. During the meeting, Snidely entered into an oral agreement with Dudley to manage Geezer's financial affairs. Snidely did not complete a client profile of Geezer. Snidely offered to review and make recommendations on Geezer's then-current living trust. Snidely prepared a Last Will, Revocable Trust and Durable Power of Attorney for management of Property and Personal Affairs, and charged Geezer $400 per hour for preparing the documents. Geezer had not requested such documents. Geezer asked Snidely to provide him with all the documents pertaining to his investments. As of the hearing date with the Disciplinary and Ethics Commission, Snidely had not provided the requested documents to Geezer. The Commission issued an Order to Revoke Permanently Snidely's right to use the CFP®, CERTIFIED FINANCIAL PLANNER™ and certification marks. The Commission ordered Snidely to verify that he was not using the marks by submitting copies of letterhead and business cards within 30 days of the Order. To comply with the Code of Ethics, before providing any services to Geezer, Snidely was required to take all of the following steps EXCEPT: A)Provide a written agreement outlining the mutually defined obligations and responsibilities of each party. B)Gather appropriate data to assess Geezer's financial situation, needs and objectives. C)Provide a written agreement outlining Snidely's compensation and potential conflicts of interest. D)Provide a written agreement outlining how and on what terms each party can terminate the agreement. E)Snidely must do all of the above.

Correct Answer: A. Although the obligations and responsibilities of each party are required to be mutually defined, this is not required to be in written form. [Rule 1.1 & 1.2] B is required by Rule 1.2 a. Since this is a financial planning engagement, C is required by Rule 2.2 e. Since this is a financial planning engagement, D is required by Rule 1.3.

A CFP® professional was asked by a client to appropriately address the following questions. 1. How much money do I need to set aside each month to send my daughter to college in fourteen years? 2. How much do I need to save so my spouse and I will have a secure retirement? 3. How much and what type, if any, of life insurance do I need? 4. What investment products would be best for me in order to execute an investment plan and asset allocation plan developed by my prior financial planner? Which of these questions considered individually would likely be considered the practice of financial planning under the Code of Ethics? A)Question 1 B)Question 2 C)Question 3 D)Question 4

Correct Answer: B Reason: The Board focuses on the specificity of the engagement to determine if it constitutes financial planning. "Answering a question of a specific nature - such as "How much money do I need to set aside each month to send my two-year-old to Notre Dame in sixteen years?" - would probably not be considered financial planning. However, answering a broader question that involves multiple aspects of a client's situation - such as "How much do I need to save so I'll have a secure retirement?" - would likely rise to the level of financial planning because of the expansiveness of the financial considerations involved." (FAQ 1-9 and FAQ 1-12). Questions 1 and 3 are much more specific than Question 2, which would generally require a more in-depth data gathering process and a broader set of recommendations. D is not correct because if the client limits the engagement with the CFP® professional to implementation activities only, the engagement may not rise to the level of financial planning. This would be the case if, for example, the recommendations provided by a third party set out an investment strategy with specific amounts allocated to specific asset classes and the CFP® professional's actions are limited to executing transactions based on the recommendations identified in the financial plan (FAQ 1-15).

John, a CFP® professional, works for a firm requiring that any investment products offered to a client be proprietary products of the firm. Jack, his client, is 55 years old and has a moderate risk tolerance. John's firm has an S&P500 index fund with a reasonable fee structure. John has discussed the fund's performance and costs with Jack and they have agreed that 60% of his equity portfolio will be allocated to this index fund. Which of the following is true according to the CFP Code of Ethics? A)John is prohibited from providing financial planning or material elements of financial planning because he may not be able to offer the client the best available option. B)John may provide financial planning or material elements of financial planning as long as the limitations concerning the proprietary products are discussed with Jack. C)John may provide financial planning or material elements of financial planning but the limitations concerning the proprietary products must be disclosed and it must be in writing to Jack D)John could enter into a limited engagement related to Jack's specific insurance needs next year with no written disclosures other than those required by regulatory bodies.

Correct Answer: C. A is not correct because the Board's definition of 'best available options' in connection with the fiduciary standard associated with a financial planning engagement allows for limitations on the certificant's recommendations (such as proprietary products) as long as they are disclosed to the client. B is not correct because although Rule 1.2 states the disclosure may be oral or written, Rule 2.2 (b) requires disclosure of conflicts of interest (which would include proprietary products) and Rule 2.2 (e) requires written disclosure for items 2.2 a-d when engaged in financial planning. C is correct since it notes that the proprietary products disclosure must be in writing, in accordance with Rule 2.2 e. D is not correct once a client becomes a financial planning client, subsequent limited engagements, would be considered a part of the continuing financial planning engagement.

Jack, a new client for Robert, a CFP® professional, requests a needs analysis concerning Jack's life insurance situation. Jack is 42, married and has two children he plans to send to college. He wants Robert to evaluate how much and what type of insurance he should purchase. Which of the following is required to be provided to Jack according to the Code of Ethics? A)A written agreement for Robert's services specifying on what terms the agreement can be terminated. B)An accurate and understandable description of the compensation arrangements being offered, in writing. C)A written summary of likely conflicts of interest between the client and the certificant. D)None of the above is required by the Code of Ethics.

Correct Answer: D A life insurance needs analysis and recommendation does not likely meet the definition of financial planning because it is focused solely on factors related to a single subject area (FAQ 1-12). Generally, the integration of two or more subject areas are needed to constitute financial planning, unless the data gathering and recommendations are in-depth and/or very broad (FAQ 1-14). Although the Board encourages professionals to provide written agreements and disclosures for limited engagements, only the disclosures listed in Rule 2.2 a-d are required, and they may be oral.

Jonathan got divorced in 2007 and subsequently had severe financial problems. In 2009, he filed for bankruptcy. After getting back on his feet, he graduated from college and got a job selling life insurance for a large national insurer. In July 2012, he completed all of the requirements for the CFP® designation. In August of 2012, he made his application to the CFP Board for licensing. Which of the following is correct under the Board's revised policy regarding bankruptcy? A)Jonathan's bankruptcy falls on the presumed unacceptable list because it is within five years preceding his application. He will be denied the right to use the marks unless he files a successful consideration request with the CFP Board. B)Jonathan's bankruptcy falls on the always bar list because it is within five years preceding his application. He will be denied the right to use the marks. C)Jonathan's bankruptcy is no longer a concern of the CFP Board as long as he discloses it in writing to all potential clients for the five year period following the bankruptcy. D)Jonathan's bankruptcy will not prevent him from becoming a CFP® professional, but it will be disclosed on the CFP® professional's public profile displayed on the CFP Board's website for 10 years.

Correct Answer: D D is correct under the revised Candidate Fitness Standards effective July 2012. His name will also be included in a press release disclosing the bankruptcy. A is incorrect because that was the prior requirement before the standards were revised. B is incorrect because two or more bankruptcies are on the presumptive bar list. C is incorrect because the Board must still be informed and will take the proactive measures listed in Item D to ensure proper disclosure. Jonathan should also make written disclosure of this event, but that alone is not sufficient.

Alice recently consulted a CFP® professional for advice regarding the federal gift tax. After establishing a planning relationship with Alice, the CFP® professional determines that Alice made the following gifts during the current year: $50,000 in payment of her niece's hospital expenses $45,000 in cash to her nephew $50,000 interest-free loan to her sister Alice asks the CFP® professional whether she is required to file a federal gift tax return for the current year. Which of the following additional information should the CFP® professional gather to answer Alice's question? Whether Alice paid the medical expenses directly to the hospital The amount of her sister's net investment income for the current year Whether Alice is married and, if so, whether she wants to elect gift splitting with her spouse A) 2 and 3 B) 1 and 2 C) 3 only D) 1, 2, and 3

D All of these statements are correct. Statement 1 is correct because if Alice paid the hospital expenses directly to the hospital, the gift is a qualified transfer and is not subject to gift tax. Statement 2 is correct because if the sister's net investment income does not exceed $1,000, the gift loan is not subject to gift tax. Statement 3 is correct because a gift tax return will be necessary if Alice elects gift splitting with her spouse. (Domain 2-Gathering Information Necessary to Fulfill the Engagement)

You are given the following information regarding three investment portfolios. Portfolio A Portfolio Return 10%, Beta 1.2, ER 12.70% Portfolio B Portfolio Return 11%, Beta 1.5, ER 15.25% Portfolio C Portfolio Return 12%, Beta 2.0, ER 19.50% Assuming a risk-free rate of return of 2.5% and a market rate of return of 11%, rank the three portfolios from best alpha to worst alpha. A) A, C, B B) C, B, A C) B, A, C D) A, B, C

D Alpha is measured as the portfolio's actual or realized return in excess of (or deficient to) the return expected by the capital asset pricing model (CAPM). Portfolio A -2.70% Portfolio B -4.25% Portfolio C -7.50% The Jensen measure concludes Portfolio A is the best portfolio on a risk-adjusted basis, even though each portfolio's alpha was a negative number. This would indicate that all three portfolio managers did not add any value to the returns of the portfolios.

This year John had the following capital gains and losses: Long-term capital gains: $12,000. Long-term capital losses: $35,000. Short-term capital gains: $80,000. Short-term capital losses: $27,000. Which of the following statements regarding the tax treatment for these gains and losses in the current year is CORRECT? A) John will recognize a $3,000 long-term capital loss, carry over the remaining $20,000 of long-term capital losses, and include $53,000 of short-term capital gains in taxable income. B) John will recognize $23,000 of long-term capital losses and $53,000 of short-term capital gains. C) John will recognize $3,000 of short-term capital loss and carry over the remaining $27,000 of capital losses. D) John will have a net short-term capital gain of $30,000.

D Because long-term capital losses exceed long-term capital gains, John has $23,000 of long-term capital losses ($12,000 LTCG − $35,000 LTCL). Similarly, John has $53,000 of short-term capital gains ($80,000 STCG − $27,000 STCL). Netting the two, John has $30,000 of net short-term capital gains.

Jon, a CFP® professional, is reviewing some of the information provided by his new client, Sara, regarding her homeowners insurance on the home she purchased 2 months ago. He discovers the home was purchased for $320,000 but has a replacement value of $300,000. The land value included in the purchase price was $30,000. Sara has a HO-3 policy insuring the home for $232,000. After his analysis is completed, he concludes Sara is underinsured. What is the amount of HO-3 coverage Jon should recommend Sara purchase for full coverage of a complete loss of her home? A) $256,000 B) $320,000 C) $240,000 D) $300,000

D For maximum coverage for a loss on the home, Jon should recommend to Sara that she increase her HO-3 coverage to the full replacement value on the home of $300,000. The price a client pays for home is an indicator of the fair market value of the home and the land but does not reflect how much it would cost to replace the home after a complete loss. Minimum coverage on the home to cover a partial loss would be $240,000 ($300,000 × 0.80) but for full coverage of a complete loss of her home, Jon should recommend a policy at the full replacement value. (Domain 3-Analyzing and Evaluating the Client's Current Financial Status)

Your clients, Edward and Mary, have a sizable estate. The couple wishes to help fund their 3-year old grandson Jimmy's college education. They have chosen not to share their plans with Jimmy's mother, Adrienne, whom you have never met. Edward and Mary wish to place the money into an account that has growth potential along with tax benefits. They consider themselves to be moderate risk takers. Which of the following statements are CORRECT? They could establish a Section 529 plan for Jimmy's benefit and fund the account with $100,000. Because Jimmy is a minor, you must contact Adrienne to get her permission before opening the account. Prior to opening an account, you should complete a thorough fact-finding interview with Edward and Mary. The couple should review any written disclosures you provide to them prior to opening a new account for Jimmy. A) 2, 3, and 4 B) 1, 2, and 3 C) 1 and 2 D) 3 and 4

D Statement 1 is incorrect. In 2018, a single contributor is permitted to make one $75,000 contribution (the gift tax annual exclusion of $15,000 times 5) to a Section 529 plan and spread that contribution over 5 years. In addition, if the contributor splits that gift with a spouse, a one-time contribution (every 5 years) of $150,000 may be made to any beneficiary. These contributions are correspondingly removed from the contributor's gross estate and are not included in the estate for estate tax purposes. Statement 2 is incorrect. Because Adrienne is not your client, you do not have to contact her regarding this planning service. Statements 3 and 4 are correct. Before providing any financial service, you should complete a thorough fact-finding interview with the couple. When offering any products or services to a client, you must provide any necessary disclosures.

Myles, a CFP® professional, specializes in life insurance planning at a medium-sized financial services firm. Last week, Mrs. Smith, age 70, came to his office to meet with him at the request of her friend, Jackie. After completing a comprehensive fact-finding interview, Myles discovered she has a net worth exceeding $3 million. Mrs. Smith told Myles that she needed a comprehensive financial plan, asset management services, and assistance in drafting estate planning documents. Even though Myles has limited experience in these areas, he decided to prepare the plan without any assistance because he wanted to keep any fees and/or commissions for himself. Also, after Mrs. Smith left his office, Myles, with her permission, contacted her estate planning attorney to discuss her situation. Finally, Myles decided to call his friend, Rex, to tell him about Mrs. Smith investments and determine how much money he could potentially earn. Based on this information, which of the following statements is CORRECT? Myles acted professionally by contacting her estate planning attorney to discuss her estate planning goals and objectives. Myles may have violated the Principle of Confidentiality by disclosing her personal information to Rex. Myles acted prudently by providing her with financial advice outside of his area of expertise. Myles could ask Mrs. Smith to loan him money at current market interest rates. A) 1, 2, and 4 B) 2, 3, and 4 C) 3 and 4 D) 1 and 2

D Statements 3 and 4 are not correct. According to Rule 3.1: A certificant shall treat information as confidential. In this case, Myles received permission to disclose her information to her attorney, but he still may have violated this rule by speaking with Rex. Myles is not permitted to borrow money from Mrs. Smith according to Rule 3.6: A certificant shall not borrow money from a client (except in certain circumstances). By providing services outside of his area of expertise and without assistance, he may have violated Rules 4.1: A certificant shall treat prospective clients and clients fairly and provide professional services with integrity and objectivity, 4.2: A certificant shall offer advice only in those areas in which he is competent to do so and shall maintain competence in all areas in which he is engaged to provide professional services, and 4.4: A certificant shall exercise reasonable and prudent professional judgment in providing professional services to clients.

Keleher and Sons, Inc., has a qualified retirement plan and must determine if its plan meets the ratio percentage test and average benefits test. There are 500 nonexcludable employees, of whom 420 are nonhighly compensated and 80 highly compensated employees. Of these, 250 of the 420 nonhighly compensated and 65 of the 80 are highly compensated benefit from the company's qualified retirement plan. Average accrued benefits for nonhighly compensated and highly compensated are 2% ($1,500) and 6% ($7,200), respectively. Which of the following statements regarding the Keleher and Sons, Inc., retirement plan is CORRECT? A) The plan does not meet either the average benefits test or the ratio percentage test. B) The plan meets both the ratio percentage test and average benefits test. C) The plan meets the average benefits test. D) The plan meets the ratio percentage test.

D The ratio percentage test is satisfied because it is greater than 70%. Ratio percentage test = (250 ÷ 420) ÷ (65 ÷ 80) = 73%. The average benefits test fails because it is less than 70%. Average benefits test = 2 ÷ 6 = 33%.

Your client, Jacob, a single taxpayer, has brought you all of his documentation for his transactions during the previous tax year. During your analysis and evaluation, you determine Jacob realized a $6,500 loss from his real estate activities as an active participant. You have already determined that his modified AGI is $130,000. What is your next step? A) In your evaluation of financial planning alternatives for Jacob, you recommend he dispose of this rental activity. B) Because Jacob's MAGI is in the phaseout range, you determine Jacob cannot deduct the loss. C) Because of the income phaseout, you note in your recommendations that Jacob may only deduct half of the loss this year. D) Having determined Jacob's MAGI, you must calculate how much, if any, of the loss is affected by the phaseout limits for the loss deduction.

D Under Practice Standard 300-1, your next step as a planner is to calculate how much of the loss is affected by the phaseout limits. Because Jacob was an active participant in the real estate activities, he may deduct up to $25,000 of real estate losses against earned income subject to the income phaseout. The phaseout begins at MAGI of $100,000 and is calculated as 50% of AGI in excess of $100,000. Jacob must reduce the maximum allowable deduction of $25,000 by 50% × ($130,000 − $100,000). Thus, the maximum deduction available to Jacob would be $25,000 − $15,000 = $10,000. Because his losses are only $6,500, he may deduct all of his losses against earned income.

Julius asks his cousin, Sam, a CFP® professional and attorney, to prepare a power of attorney for him. Sam agrees and meets with him when the documents are completed. During that meeting, Sam presents an investment plan for Julius based on conversations he and Julius have had over the years. Julius thanks Sam for preparing the power of attorney and pays him a fee. Julius tells Sam he is not currently interested in an investment, but may contact him in the future if circumstances change. Based on the facts provided, is Sam providing financial planning for Julius? A) No, because Sam is a member of Julius's family B) Yes, because Sam charged Julius a fee for his services. C) Yes, because Sam created an investment plan for Julius and investment planning is one of the financial planning subject areas. D) No, because Julius did not ask Sam to prepare, nor did he expect Sam to create an investment plan for Julius.

D) No, because Julius did not ask Sam to prepare, nor did he expect Sam to create an investment plan for Julius. Julius expected no more from Sam than the preparation of the power of attorney; he did not request any of the financial planning subject areas to be addressed or any recommendations to be made. The fact that Julius paid a fee for Sam's services and the fact that the parties are related does not make the assistance Sam provided financial planning. Even though investment planning is a financial planning subject area, the plan was not requested by Julius, nor is it certain that Sam did enough data gathering to be able to provide sound recommendations. (Domain 8-Practicing Within Professional and Regulatory Standards)

Ashely, a CFP® professional, had recently relocated her office to a new building in her city's financial district. She believed this location would help her market her business in a more professional manner. She found the move to be quite overwhelming and she was arrested for driving while intoxicated (DWI). After careful consideration, she pled guilty to a charge of DWI and bail jumping in court this morning. In addition, she had difficulty being properly prepared for her first client meeting this afternoon. After the meeting, she discovered her client did not adequately complete her firm's risk-profile questionnaire. Ashley felt she could still prepare the client's financial plan even though some critical information was missing on the questionnaire. Finally, she just remembered that her CFP® certification was up for renewal soon and she did not have the required number of continuing education hours. Which of the following is (are) required of Ashley? She has 45 days to report her guilty plea to CFP Board. She has 30 days to report her change of address to CFP Board. She should contact her client to request the missing information before proceeding with completing a financial plan. She must complete any continuing education requirements to retain the right to use the CFP® marks. A) 1 only. B) 2, 3, and 4 C) 1, 2, and 3 D) 3 and 4

D. Statements 1 and 2 are not correct. The Disciplinary Rules and Procedures were updated to reflect a 30 calendar day reporting requirement for criminal matters, such as her guilty plea. Also, she may receive a suspension to use the CFP marks for the DWI and bail jumping. According to Rule 6.3: A certificant shall notify CFP Board of changes to contact information, including, but not limited to, email address, telephone number(s) and physical address, within forty-five (45) days. According to Rule 3.3: A certificant shall obtain the information necessary to fulfill her obligations. If a certificant cannot obtain the necessary information, the certificant shall inform the prospective client or client of any and all material deficiencies. According to Rule 6.2: A certificant shall meet all CFP Board requirements, including continuing education requirements, to retain the right to use the CFP® marks.

Which of the following statements is(are) CORRECT? Fee simple rental property owned by the decedent and the decedent's half interest in community property owned at death are included in the probate estate. A Section 529 plan and a Section 2503(b) trust both require all income to be distributed annually to the beneficiary. Losses on Section 1244 stock gifted to a minor child are limited to $25,000 annually because of the child's single filing status. A trust beneficiary receives a Schedule K-1 summarizing the amounts and character of taxable income distributed.

D. Statement 1 is correct. Statement 2 is incorrect. Only the Section 2503(b) trust requires income to be distributed to the beneficiary annually. A Section 529 plan is a qualified tuition plan and contributions and earning are generally used to pay qualified tuition expenses, tax-free to the beneficiary. Statement 3 is incorrect. A single taxpayer, including a minor, is allowed a maximum $50,000 Section 1244 ordinary loss deduction regardless of the taxpayer's age. Statement 4 is correct.

Lisa is a condo owner and has an HO-6 policy. She purchased the condo for $400,000. Her HO-6 policy is an open peril policy and has a face value of $360,000. Her contents are covered on a named peril basis with $100,000 in coverage. She also has an 80% coinsurance requirement. A tornado hits the building and completely destroys the roof of the condo. The cost to repair the roof is $50,000. How much would her condo policy cover for the roof damage?

The correct answer is "A." An HO-6 policy does not provide coverage for the building or roof. The building and roof are covered by the condo association policy, which covers all exterior walls and roof. The HO-6 policy covers all interior walls for a condo.

Dave is 46, married and has an annual salary of $60,000. His employer offers group term life insurance coverage equal to 2 times his annual salary. The employer's cost for Dave is $.40 per $1,000 of which Dave pays $.15 per month per $1,000. The Table 1 (Section 79) rate for 45-49 year olds is $0.29 per $1,000. What additional income must Dave include in his taxable income this year resulting from the group term insurance? Round your answer to the nearest dollar. A)$28 B)$126 C)$210 D)$244

The correct answer is "A." Dave is paying $216 each year for the coverage ($120 x 0.15 x 12). The Table I cost is calculated by subtracting $50,000 (the tax-free amount allowed under Section 79) from the $120,000 actually purchased, dividing the remainder by $1,000, multiplying the Table 1 rate of 0.29 times 12. ($120,000 - $50,000 / $1,000 x 0.29 x 12). So, the Table 1 premium is $244 (rounded.) Subtract the $216 already paid by Dave from the $244 Table 1 premium to determine the additional taxable income ($244 - $216 = $28).

Which of the following correctly describes "eligible individual accounts"? I. May not invest plan assets in qualifying employer securities. II. Include defined contribution plans. III. Are subject to Pension Benefit Guaranty Corporation (PBGC) coverage and insurance premiums. IV. Are subject to minimum funding requirements. A)II only. B)II and IV only. C)I only. D)I and III only.

The correct answer is "A." Eligible individual accounts (usually associated with profit sharing, 401(k) and ESOP) are defined contribution plans, and are not subject to PBGC, nor minimum funding requirements.

A successful architect wants to purchase disability income insurance. She is concerned about becoming totally disabled, but also about a reduction in income if she is obliged to reduce her workload because of a less-than-total disability. To satisfy these concerns, which of the following should be included in her disability income coverage? A)Residual disability benefits. B)A change-of-occupation provision. C)Dismemberment benefits. D)A relation of earnings-to-insurance provision.

The correct answer is "A." Residual benefits cover partial disability and directly address the concern that the client has expressed. Options "B," "C" and "D" are valid provisions, but do not in any way address the client's area of concern.

Under the terms of a divorce agreement dated 1/3/18, Larry is required to pay his wife Joyce $2,100 per month in alimony for a minimum period of 10 years and $300 per month in child support. For a twelve-month period, Larry can deduct from gross income (and Joyce must include in gross income): A)$0 B)$25,200 C)$3,600 D)$28,800

The correct answer is "A." The $300 per month for child support is not deductible by Larry. Child support payments are not deductible to the payor nor includable to the payee. Larry's $300 per month in child support will remain part of his gross income. The $2,100 is not alimony since it could extend beyond her death as the required payment is for a minimum of 10 years.

The optimum portfolio is said to occur at the point of tangency of which of the following two measures? A)Indifference curve and efficient frontier. B)Standard deviation and median expected return. C)Beta coefficient and portfolio data. D)Covariance and correlation coefficient.

The correct answer is "A." The Indifference Curve is the risk return trade-off which investors are willing to make, while the Efficient Frontier is the best possible returns that could be expected from all possible portfolios. At the point of tangency, one has attained the optimal portfolio.

John Risotto has a cash need at the end of nine years. Which of the following investments best meets this need and serves to immunize the portfolio initially? I. An 11-year maturity coupon bond. II. A 9-year maturity coupon Treasury note. III. A series of Treasury bills. A)I only. B)II and III only. C)II only. D)I and II only.

The correct answer is "A." The process of portfolio immunization entails not maturity of a security, but its duration. Duration is based on coupon rate. The larger the coupon payment, the shorter the duration. This being the case, a bond generally pays higher interest than a note, and a note pays higher than short-term Treasury bills. Given this information, one could reasonably expect a shorter duration (than time to maturity), while receiving better immunization from the bond.

Which of the following statements accurately reflect the characteristics of a Section 457 plan? I. Benefits taken as periodic payments are treated as ordinary income for taxation. II. Lump-sum distributions are eligible for 5-year and/or 10-year averaging. III. Deferred amounts are subject to Social Security and Medicare taxes at the later of: performance of services or employee becomes vested in the benefits. IV. Income tax withholding is not required until funds are actually received, not constructively received. V. Cannot exceed the smaller of $18,500 or 100% includible compensation. A)I, III and V only. B)II, IV and V only. C)I, II, IV and V only. D)I, II, III, IV and V.

The correct answer is "A." There are no special tax advantages provided for 457 plans distributed in a lump-sum. Income tax withholding is required once the benefits are constructively received, even if not actually received.

Your client Bebe Rebozo is contemplating the exchange of two parcels of investment land for two similar parcels in two separate transactions. Given the following details of the proposed transaction, compute the amount of recognized gain and loss (if any) on both parcels if your client completes the exchanges: Parcel A: Ten acres of land acquired 15 years ago with a current basis of $50,000. In exchange your client will receive eight acres of land (FMV = $80,000) and $20,000 in cash. Parcel B: Twenty acres of land acquired two years ago with a current basis of $100,000. In exchange, your client will receive twelve acres of land (FMV = $75,000) and $10,000 in cash. A)Parcel A Recognized Gain = $20,000; Parcel B Recognized Loss = $0 B)Parcel A Recognized Gain = $20,000; Parcel B Recognized Loss = $10,000 C)Parcel A Recognized Gain = $50,000; Parcel B Recognized Loss = $10,000 D)Parcel A Recognized Gain = $20,000; Parcel B Recognized Loss = $15,000

The correct answer is "A." This question pertains to like kind exchanges where boot is involved. The rule is that any realized gain will be recognized to the extent of the lesser of realized gain or boot received. In this case, there was a realized gain of $50,000 ($50,000 basis for $100,000 market value). The boot of $20,000 is recognized as gain since it is the lesser of boot or realized gain. Parcel B will have no gain in that there is no realized gain between the basis of the property given up and the fair market value of the property received. There is a realization but it is not recognized. Losses in like kind exchanges are not recognized.

Five months ago, Sammy Free inherited property from his uncle's estate. The value of the property received by Sammy was declared in the estate to be valued at $20,000. Sammy titled the property in Community Property with his spouse, Alline. Sammy died last month when the property was valued at $50,000. Sammy's will left everything to his spouse, Alline. Within weeks, Alline sold the property for $55,000. What is her gain or loss at the date of the sale? A)A $5,000 long-term capital gain. B)A $35,000 long-term capital gain. C)A $20,000 long-term capital gain. D)A $35,000 short-term capital gain.

The correct answer is "A." When one spouse dies, both halves of the community property receive a "stepped-up" income tax basis. While his original basis was $20,000, her basis in her half of the community property is $25,000 and since she also inherited his half of community property her total basis at the time of the sale was $50,000 therefore the gain is $5,000 and because it was inherited property the gain is long-term.

Danny is starting a new business. He is concerned about liability. He would like to have flow-through taxation. At some point, he would like to be able to easily sell interests in the business, but he does not expect to have more than 20 investors. Danny does not want to pay self-employment taxes on all income. Which of the following entities would best suit Danny's needs? A)A Proprietorship. B)A S corporation. C)A C corporation. D)A Partnership.

The correct answer is "B". Option "A" is not correct because a proprietorship does not provide limited liability and Danny would need to change entities to take on future investors. Option "C" is not correct because a C coporation would not provide flow-through taxation. Option "D" is not correct because a partnership would not provide limited liability and Danny would have to pay self-employment taxes on the business net income. Therefore, only option "B" meets all of Danny's requirements.

The Health Insurance Portability and Accountability Act of 1996 (HIPAA) impacts an employee and employer in which of the following ways: I. An employee without creditable coverage can generally only be excluded by the group health insurance plan (if offered) for up to twelve months. II. The waiting period is reduced by the amount of "creditable coverage" at a previous employer. III. If the employee does not enroll in the group health insurance plan at the first opportunity, an 18-month exclusion period may apply. A)I and II only. B)I, II and III only. C)II and III only. D)II only.

The correct answer is "B." All three statements are true. If you have a pre-existing condition that can be excluded from your plan coverage, then there is a limit to the pre-existing condition exclusion period that can be applied. HIPAA limits the pre-existing condition exclusion period for most people to 12 months (18 months if you enroll late), although some plans may have a shorter time period or none at all. In addition, some people with a history of prior health coverage will be able to reduce the exclusion period even further using "creditable coverage." People with a history of prior health coverage will be able to reduce the exclusion period even further using "creditable coverage."

Your client, John Kent, purchased a limited payment whole life policy 15 years ago. He would like to stop paying the premiums on his policy, but continues to need the same amount of insurance. If he did so, which one of the following is a non-forfeiture option he could use? A)Reduced paid-up insurance. B)Extended term insurance. C)Installments for a fixed period. D)One-year term.

The correct answer is "B." An extended term insurance is correct because extended term insurance is the only choice that is a non-forfeiture option. Option "A" - Although this is a non-forfeiture provision, the amount of insurance coverage would be reduced. Option "C" is a settlement option, and Option "D" is a dividend option.

Which of the following is true concerning IRA contributions? A)An employee who makes voluntary contributions to a 401(k) plan is not considered an active participant. B)An employee who receives no contributions or forfeiture allocations in their employer's profit sharing plan is not considered an active participant. C)An employee who makes no voluntary contributions to a thrift plan yet receives forfeiture allocations to a profit sharing plan is not considered an active participant. D)An employee participating in a Section 457 plan is considered an active participant if employee pretax deferrals are elected.

The correct answer is "B." Answers "A" and "C" are conditions of being considered an active participant. Answer "D" is incorrect because 457 plan participants are not considered active participants for IRA contribution purposes.

Which of the following is true regarding qualified incentive stock options? I. No taxable income will be recognized by the employee when the qualified option is granted or exercised. II. The income from sale of the qualified option will always be taxed as capital gains when the stock is sold. III. The income from sale of the qualified option will be taxed as ordinary income regardless of when the stock is sold. IV. The employer will not be able to deduct the bargain element of the option as an expense under any circumstance. V. For favorable tax treatment the option must be held two years and the stock for one year after exercise. A)II and IV only. B)I and V only. C)III only. D)I, II and V only.

The correct answer is "B." In Statement "II," be careful of "always"! In Statement "III," if held longer than one year, they receive capital gains treatment. In Statement "IV," under most circumstances, the bargain element is deductible. There are exceptions when certain qualifications have not been met for deductibility, such as time employed, time to exercise in excess of rules, etc.

Your client has purchased stock with a margin position that required 50% initial margin and a 35% maintenance margin. The stock was originally valued at $23 per share when the transaction was undertaken and your client bought 1,000 shares. What stock price will trigger a margin call? A)$13.04 B)$17.69 C)$26.45 D)$32.86

The correct answer is "B." Price = Loan ÷ (1 - MM) = (23 x. 50) ÷ (1 - .35) = 17.69

Risk transfer (or the use of insurance) would best be utilized in the case of: A)High frequency / high severity. B)Low frequency / high severity. C)High frequency / low severity. D)Low frequency / low severity.

The correct answer is "B." Risk transfer (or the use of insurance) would be utilized in the case of low frequency / high severity. We use insurance to transfer risk for life, income and property. All areas we hope to have no losses, but if we did, they would alter our way of living. Low Frequency/Low Severity = Retention Low Frequency/High Severity = Insurance(Transfer) High Frequency/Low Severity = Reduction, non-insurance transfer High Frequency/High Severity = Avoidance (catastrophic risk)

An insured gave his correct age as 45, but when it was written down by the agent, the numbers were transposed presenting the insured as age 54 on the application. The insured purchased and was approved for the insurance and paid regular premiums. One day after three years of ownership, the insured came across the policy and noticed the error. He called the company. What was the company most likely to do? A)Cancel the policy and take the insured to court for fraud. B)Provide the insured with a policy representing the face amount that the premiums would have purchased. C)Give the insured back the difference with interest. D)Remove excess amounts from the interpolated required reserve and place them in the general fund.

The correct answer is "B." The insured would be provided with a policy that represented the amount of life insurance the premium would have purchased at the younger age of 45.

Jerry Rivers owns a $250,000 level-term life policy which he purchased five years ago. He has paid premiums of $400 per year for the past five years. He also owns a $125,000 whole life policy which he purchased fifteen years ago. He has paid premiums of $2,000 per year for the past 15 years, and now the policy has a cash surrender value of $40,000. Over the years, the whole life policy has paid cash dividends to Jerry. The cumulative dividends paid to Jerry since inception totals $5,000. Jerry has decided to cancel his $125,000 whole life policy. Which statement is true? A)Jerry has a taxable gain of $10,000. This gain will be treated as a long-term capital gain. B)Jerry has a taxable gain of $15,000. This gain will be treated as ordinary income. C)Jerry has a taxable gain of $70,000. This gain will be treated as ordinary income. D)Jerry has NO taxable gain.

The correct answer is "B." Upon surrendering his whole life policy, Jerry received $40,000 cash value where he had paid only $30,000 - $5,000 (dividends) = $25,000 (basis). $40,000 - $25,000 = $15,000 is treated as ordinary income, taxable in the year it is received.

You are faced with several fixed income investment options. Which of these bonds has the greatest interest rate risk? A)A U.S. Treasury bond with an 11.625% coupon, due in five years with a price of $1,225.39 and a yield to maturity of 6.3%. B)A U.S. Treasury strip bond (zero-coupon) due in five years with a price of $735.12 and a yield to maturity of 6.25%. C)A corporate B-rated bond with a 9.75% coupon, due in five years with a price of $1,038.18 and a yield to maturity of 8.79%. D)A U.S. T-bill selling for $950 due in six months.

The correct answer is "B." With the term being equal, the bond with the lowest coupon will have the biggest duration. The bigger the duration, the price more sensitive the bond is to interest rate changes. Bond B has the lowest coupon, zero.

Hannah owns an event planning company that specializes in very high-end events. Several years ago, Hannah purchased a magnificent chocolate fountain for $3,000 and has since taken $1,200 in depreciation deductions on the fountain. Hannah is now ready to replace the fountain with tools for creating ice sculptures, but she is not sure what the tax consequences of selling the fountain will be. Which of the following statements is true regarding the tax consequences of selling the fountain? A)If Hannah sells the chocolate fountain for $1,800, she will have a $1,200 ordinary loss. B)If Hannah sells the chocolate fountain for $1,700, she will have a $100 capital loss. C)If Hannah sells the chocolate fountain for $2,000, she will have an ordinary gain of $200 and no capital gain. D)If Hannah sells the chocolate fountain for $3,300, she will have a $1,500 capital gain.

The correct answer is "C". Option A: Amount Ralized = 1800, Adjusted Basis = 1800, Gain/Loss = 0, Tax Impact = None, Economic Reality = Depreciation estimate was perfect Option B: Amount realized =1700, adjusted basis = 1800, loss= 100, Tax impact = Ordinary loss, economic reality= taxpayer took too little depreciation Option C: Amount realized=2000, adjusted basis = 1800, gain = 200, Tax impact = Ordinary gain, Economic reality= taxapayer took too much depreciation Option D: amount realized = 3.3k, adujsted basis=1.8k, gain=1500, tax impact = Part ordinary gain (1200), economic reality = taxpayer took too much depreciation and asset appreciated

Under which of the following circumstances is a trip outside the United States considered to be purely for business? I. The taxpayer does not have control over the timing or arrangements for the trip. II. The trip outside the United States lasts for less than seven days. III. Less than 50 percent of the time spent on the trip was personal. IV. Vacation was not a primary consideration for the trip. A)I only. B)II and III only. C)I, II and IV only. D)I, II, III, and IV

The correct answer is "C". A trip outside the United States is considered to be purely for business when less than 25 percent of the time spent on the trip was personal. All of the other statements regarding travel outside the United States are correct.

Tony Scarponi has come to you asking about the basis of property that his brother Calvin gave to him. The property had a market value of $75,000 and Calvin's adjusted basis in the property was $18,000 at the time of the gift. Calvin paid gift tax of $3,500 on the gift. Tony wants to know what his adjusted basis in the property is. Assume Calvin had utilized his annual gift tax exclusion for gifts previously given to Tony that year. What will you tell him? A)Tony's new basis is $18,000, the same as Calvin's basis was at the time the gift was made. B)Tony's new basis is the fair market value of the gift at the time of the gift. C)The adjusted basis for Tony is $20,660. D)The adjusted basis for Tony is $21,500.

The correct answer is "C." Increase in Donee's Basis = (Appreciation of the Property/ Taxable Gift) x Gift Tax Paid FMV of Property at Date of Gift [($57,000 ÷ $75,000 = .76) x $3,500] + $18,000 = $20,660

You have recommended a growth mutual fund to a new client. The client considered your recommendation and asked why he shouldn't invest in another fund that he had been following, which appeared to have a better performance over the past three years. You explained the concept of risk-adjusted performance and obtained the following information about the two funds: Your fund: Three-year total return = 13.5% Average P/E ratio = 20 Standard deviation = 19 Beta = 1.03 Client Fund: Three-year total return = 14.74% Average P/E ratio = 24 Standard deviation = 23 Beta = 1.24 Using Treynor, which fund would you recommend if the risk-free return is 3%? A)Client fund, because its Treynor is higher than your fund. B)Client fund, because its Treynor is lower than your fund. C)Your fund, because its Treynor is 10.2% compared with the client's Treynor of 9.4%. D)None of the above.

The correct answer is "C." Your Fund: T = (13.5 - 3.0) ÷ 1.03 = 10.1942 Client Fund T = (14.7 - 3.0) ÷ 1.24 = 9.4355

Gina, age 79, recently had a stroke. Afraid that she may not live long enough to see her family enjoy it, she would like to transfer the beach house she owns to her daughter Taylor. While Gina is willing to make the transfer gratuatious in whole or part, Gina does not want to pay any gift tax or utilize any of her lifetime credit amount. Which of the following techniques, if used by Gina to transfer the beach house to Taylor, will not result in a taxable gift? A)GRAT. B)QPRT. C)SCIN. D)GRUT.

The correct answer is "C." A SCIN is a note with a self cancelling premium payment attached so that the note will cancel at the transferor's death. The GRAT, QPRT and the GRUT are irrevocable trusts and would result in a current taxable gift.

The holding period of property acquired by gift may begin on: A)The date the property was acquired by the donor only. B)The date of gift only. C)Either the date the property was acquired by the donor or the date of the gift. D)Some other date.

The correct answer is "C." A person receiving a gift has a holding period of the donor plus the donee if at disposition he uses the gain basis. However, the holding period for a gift utilizing the loss basis (where a double basis rule applies) starts the holding period at the date of the gift.

A client with a large, well-diversified common stock portfolio expresses concern about a possible market decline. However, he/she does NOT want to incur the cost of selling a portion of their holdings NOR the risk of mistiming the market. A possible strategy for him/her would be: A)Buy an index call option. B)Sell an index call option. C)Buy an index put option. D)Sell an index put option.

The correct answer is "C." A put option index that closely mirrors the client's portfolio will allow for minimization of loss in the event of market decline.

Which of the following accurately reflect(s) the taxation of an annuity owned by a decedent? A)The decedent's will determines who receives proceeds of annuity because beneficiary designation become null and void at death. B)Accumulated interest is income tax exempt. C)Deferred interest is subject to Income in Respect to a Decedent (IRD) D)The annuity proceeds must always be probated.

The correct answer is "C." Annuities are a contract and as such avoid probate and pay proceeds to a named beneficiary. Accumulated interest income is taxable. IRD is estate and income taxable, but the income tax will be offset by estate tax paid.

Making arrangements to deal with the possibility of physical or mental incapacity is an important area of estate planning. Which of the following arrangements may be used to deal with unexpected incapacity? I. Springing durable power of attorney. II. Revocable living trust. III. Fee simple titling. IV. Living will. A)I and II only. B)II and IV only. C)I, II and IV only. D)I, II, III and IV.

The correct answer is "C." Fee simple ownership is not an arrangement that helps to deal with unexpected incapacity. All of the other arrangements are methods of dealing with unexpected incapacity.

While John is working on his garage roof, he slips, falls off the roof, and lands on the driveway next to the car. John broke his arm in the fall. He should seek to collect and will be successful in doing so from which of his insurance policies? A)Coverage F, medical payment insurance on his homeowners. B)Medical pay on his auto insurance. C)His personal health insurance policy. D)His extended coverage on his life insurance policy.

The correct answer is "C." Option "A" - Homeowners does not pay for injury to the insured. Option "B" - Medical pay auto pays for the insured if he or she is injured "in, on, or about the covered auto." Here he was close, but not close enough. His personal health insurance (Option "C") will pay.

Marsha has the following income and losses for the current year: I. ($1,000) loss from a 30% interest in Laminate Partnership in which she does NOT materially participate. II. ($1,500) loss from a 2% limited partnership interest in Venture, a limited partnership. III. ($3,000) loss from a 12% interest in an S corporation in which she manages one of the departments. IV. $40,000 salary as manager with an S corporation. V. $1,200 of dividend income from Higher Mutual Fund. Assuming Marsha has sufficient at risk basis in each of the entities, what is Marsha's adjusted gross income? A)$35,700 B)$41,200 C)$38,200 D)$36,700

The correct answer is "C." Option "I" - A loss from a limited partnership in which there is no material participation is governed under the passive activity loss rules. Since there is no other passive activity income to offset the loss, the loss is not currently deductible. Option "II" - The same passive activity loss rules apply, and therefore, the loss is not currently deductible. Option "III" - Because she is a material participant in managing the S corporation, the losses are deductible. Option "IV" - Wages are always included in AGI. Option "V" - Dividend income unless excluded is included in AGI. $40,000 (wages) minus $3,000 (S corp loss) plus $1,200 (dividends) = $38,200.

Identify the statement(s) below that correctly characterize(s) property interests held by the decedent that, at death, pass by operation of law: I. If the property passes by operation of law, the property avoids probate. II. If the property passes by operation of law, it will not be included in the decedent's gross estate. III. Property that passes by operation of law cannot qualify for the marital deduction. IV. For property passing by operation of law, the titling on the instrument determines who will receive the property. A)I only. B)II, III and IV only. C)I and IV only. D)I, III and IV only.

The correct answer is "C." Property passing by operation of law will not be probated, but will not exclude the property (or at least a portion of it) from inclusion in the gross estate. Property passing by operation of law qualifies for the marital deduction if it is passed in a qualifying way to the spouse.

A client sold an apartment building last year for $100,000, paying a sales commission of $5,000 plus $2,500 closing costs. The building originally cost $80,000 20 years ago. Total straight line depreciation of $40,000 had been taken. The building had a mortgage of $60,000 which was assumed by the buyer. What is the seller's adjusted cost basis prior to the sale? A)$32,500 B)$37,500 C)$40,000 D)$42,500

The correct answer is "C." The adjusted cost basis is determined by subtracting from the acquisition costs any depreciation or unrecovered casualty losses. The adjusted cost basis was as follows: $80,000 (acquisition cost) minus $40,000 (depreciation) = $40,000 adjusted cost basis. D)

Barbara Reed owns an LMN, Inc. bond with a par value of $1,000. LMN is a AA-rated bond that matures in 7 years. Barbara receives $55 of interest income from LMN semiannually. Comparable debt, i.e., is AA-rated, 7-year maturity, yields 12%. The bond's duration is 5 years. What is the intrinsic value of the bond? A)$703.36 B)$880.80 C)$953.53 D)$954.36

The correct answer is "C." The intrinsic value of a bond is its calculated present value. N = 7 x 2 i = 12 ÷ 2 PV = ? PMT = 55 ($110 ÷ 2) FV = 1,000

A client has bought a stock for $40 per share. At the end of the first year, she purchases another share at $43 per share. At the end of the second year with the share price of $48, she sells her shares. Along the way, at the end of each year, she received a $2 per share dividend. What is the time-weighted return on her investment? A)9.53% B)13.5% C)14.3% D)16.6%

The correct answer is "C." This is simply an uneven cash flow problem. CF0 = <$40> CF1 = $2 CF2 = $50 IRR = 14.33% Note: Since this is a time weighted return, we are only concerned about the security's cash flow. Therefore, we ignore the second purchase at $43 per share.

What is the return that your client should expect from a security that last year returned 11.7% with a standard deviation of .146, a beta of 1.2, when the overall market return has been 10.93%, and U.S. Treasury issues are currently delivering around 3.56%? A)14.6% B)13.3% C)12.4% D)11.7%

The correct answer is "C." Using the CAPM one can calculate this answer. Standard deviation and last years return are merely distractors. ER = Rf + B (Rm - Rf) ER = .0356 + 1.2 (.1093 - .0356) ER = .0356 + .0884 ER = .1240 = 12.4%

A will can accomplish which of the following estate planning objectives? A)Avoids probate. B)Provides for decisions in the event of incompetency. C)Can establish a testamentary credit shelter trust. D)Can override a beneficiary designation on a qualified retirement plan.

The correct answer is "C." Wills are always probated. Incompetency management must be addressed in a separate document, such as a trust or power of attorney. Contract named beneficiaries take trump over will stipulations. Business price or value is determined either in an appraisal or in a buy/sell agreement.

Smith and Jones purchased a warehouse. The value of the warehouse was $1 million. Their insurance agent carefully explained the coinsurance requirement of 80% to them. Smith and Jones wisely decided to insure the warehouse for the full value that they had paid for it. Their agent, however, had failed to mention the inflation protection option which would have upped the amount of insurance they had each year for coverage on the warehouse. They also had a $5,000 deductible. As a result of all this, at the end of five years with the warehouse value approximately $1,300,000, Smith and Jones had a fire. The fire caused $50,000 worth of damage. The insurance company paid the following: A. 50k B. 45k C. 43,077 D. 48,077

The correct answer is "C."Amount carried ($1,000,000) divided by amount required (80% of $1,300,000 which is $1,040,000) times the loss, minus the deductible of $5,000. The answer is $43,077. If the inflation option had been chosen, then the coverage would have increased with inflation and therefore would have met the coinsurance requirement. In that case, the insured would have only paid the deductible.

Which of the following statements accurately describes a situation where the use of a Flexible Spending Account (FSA) would be advisable? I. The employer's medical plan has large deductibles or large coinsurance or copayment provisions. II. There is a need for benefits that are sometimes difficult to provide on a group basis, such as dependent care. III. The employees are primarily non-union and operating outside of a collective bargaining agreement. IV. There are a great many employees who have an employed spouse with duplicate medical coverage. A)I and II only. B)I, III and IV only. C)I, II, and III only. D)I, II, III and IV.

The correct answer is "D." All of the choices listed here are reasons why an employer may want to offer an FSA.

The Generation Skipping Transfer Tax (GSTT) has all the following characteristics, except: A)GST outright gifts qualifying for the annual exclusion are not subject to the tax. B)Assets transferred to a trust that has a grandchild as the sole beneficiary may be subject to both gift and generation skipping transfer tax. C)If all the children of a trust are grandchildren (whose parents are living) of the grantor then the trust is subject to GSTT. D)A "skip person" is a person who is one or more generations younger than the grantor.

The correct answer is "D." Answers "A" through "C" are true, but in the case of "D," a grandchild whose parent has died has moved up a generation with regard to skip-person considerations. A skip beneficiary is generally a person who is two or more generations younger than grantor.

What effect does claiming the Section 179 deduction have on an asset's adjusted taxable basis? A)Immediate expense of the total cost and a resulting basis of zero not subject to recapture. B)Immediate expense of the total cost and a resulting basis of zero subject to recapture. C)Partial expense with a reduced basis and not subject to recapture. D)Any amount of expense not to exceed the total cost, subject to income and Section 179 expense limits, reduces the adjusted taxable basis.

The correct answer is "D." Claiming Section 179 expense immediately reduces the basis of the property by whatever the amount claimed (not to exceed original cost). The amount of deduction is limited in total and further subject to income limitation before the deduction is taken.

Which of the following legal requirements apply to Employee Stock Ownership Plans (ESOPs)? I. ESOPs must permit participants, who are aged 55 or older and who have at least 10 years of service, the opportunity to diversify their accounts. II. ESOPs can be integrated with Social Security. III. An employer's deduction for ESOP contributions and amounts made to repay interest on an ESOP's debt cannot exceed 25% of the participant's payroll. IV. The mandatory 20% income tax withholding requirement does not apply to distributions of employer stock from an ESOP. A)I and II only B)II and IV only C)I, II and III only D)I and IV only

The correct answer is "D." Deductions for interest payments are not limited for ESOP plans. Deductions for repayment of principal is limited to 25% of covered compensation.

In step two of the financial planning process (establishing facts, information and data), all of the following are qualitative data except: A)Client health status. B)Any potential changes in current lifestyle. C)Risk tolerance levels. D)Assets and liabilities.

The correct answer is "D." In step two of the financial planning process, "establishing facts and gathering information", assets and liabilities are quantitative data. Names and numbers are usually indicators of quantitative data.

Marie is the founder and sole owner of Purple Cakes Bakery. Allen has offered to buy her business for a price Marie considers reasonable, but Allen does not have all of the funds necessary to pay for the business at the current time. Marie is in good health, her true life expectancy is much greater than the IRS life expectancy factor, and she wants to accept Allen's offer. Allen is not related to Marie and has good credit. Given these facts, which transfer method should be used to transfer the business to Allen? A)Grantor Retained Annuity Trust. B)Self-Cancelling Installment Note. C)Private Annuity. D)Installment Sale.

The correct answer is "D." Marie would sell the business to Allen utilizing an installment sale and would charge a reasonable rate of interest. Because Allen would not have to pay the full sale price at the date of the transfer, he would not need to have all of the funds necessary at that time. Because Allen is not related to Marie, she would not have any reason to enter into a GRAT, SCIN, or Private Annuity, which may inequitably benefit Allen. The best situation would be for Marie to sell the business to Allen in an outright cash sale, but that is not an option in this problem.

Suzanne York has a personal residence that she wants to pass to her children upon her death. Rather than waiting, she gives the children the home with the stipulation that she can continue to live in the home for the rest of her life. What best describes the transaction? A)A reversionary interest. B)A life interest. C)A term interest. D)A remainder interest.

The correct answer is "D." She has made a gift with a remainder interest. Reversionary interest would have the home ownership returning to her. A life interest would be a controlling interest for life, and term interest would be a limited time.

Pat established his business one year ago. He has hired two assistants. He would like to establish a retirement benefit plan for himself and his two assistants, who want to make voluntary contributions. He is concerned about cash flows for unforeseen business obstacles and future expansion. Which of the following reasons is/are appropriate to recommend the establishment of a retirement plan? I. A retirement plan would allow Pat to save for his own retirement. II. Tax savings would help to offset the cost of employer contributions. III. A retirement plan would give the appearance of business stability and would be an asset in the securing of business loans to meet growth and cash flow needs. A)I only. B)II only. C)I and III only. D)I and II only.

The correct answer is "D." Statement "III" is incorrect because a retirement plan cannot be used as collateral for a loan made to the plan sponsor. This would be a prohibited transaction.

Your client wants to accomplish the following: - Provide inflation protected income stream for parents. - Reduce income taxes. - Fulfill charitable intent. - Provide some control over the assets. Which of the following trust or funds would accomplish your client's goals? A)A charitable lead unitrust. B)A charitable remainder annuity trust. C)A charitable pooled income fund. D)A charitable remainder unitrust.

The correct answer is "D." The CRUT accomplishes all of these objectives. Income for parents eliminates "lead trust" and "pooled income." Control over assets eliminates CRATs. CRAT cannot provide for inflation. CRUT income can increase.

XYZ Corporation is a closely held corporation. Martin McFly, along with the three other owners, set up a stock redemption agreement requiring the corporation to buy all shares of a deceased or disabled shareholder. The plan is funded by entity life insurance policies on each shareholder. Premiums are paid by the corporation. The agreement states that the share price will be established by an independent, competent third party appraiser. What are the tax implications of this plan? I. A deceased shareholder's gross estate will be increased by the amount of the life insurance. II. There is no step-up in basis for decedent's family on the shares of stock covered by the plan. III. The corporation will owe income tax on the difference between the cash value of the policy and the death benefit amount. A)I, II and III only. B)I and III only. C)II only. D)None of the above.

The correct answer is "D." The deceased shareholder's estate will not increase due to the life insurance, as the deceased shareholder does not own the policy and already has the value of his interest in his gross estate. There is a step-up in basis because the decendant died and the shares are "purchased" by the corporation. The corporation is "owed" the premiums by the individual at death and does not pay tax.

Your client, ABC Corporation, is considering adopting some form of retirement plan. The client states objectives for a plan to be, in the order of importance: I. Rewarding long-term employees. II. Retention of employees. III. Providing a level of income at retirement equal to 50% of an employee's earnings. IV. Tax-deductible funding. V. No risk to employees of benefits available. The company indicates it is willing to contribute an amount equal to 30% of payroll to such a plan. The company has been in business for 22 years, and during the past decade has consistently been profitable. They furnish you with an employee census. Based upon the stated objectives, you advise ABC Corporation that the most suitable retirement plan for the corporation would be: A)Money purchase pension plan. B)Non-qualified deferred compensation plan for long-term employees. C)Combination of defined benefit and 401(k) plan. D)Defined benefit plan.

The correct answer is "D." The only plan which meets all the criteria is the defined benefit plan. All other answers would not meet criteria "III" and/or "V".

Mary is a CFP® professional and is in the analyzing and evaluating step of the financial planning process. Mary is developing a capital needs analysis for her client and has established assumptions for tax rates, investment returns and inflation rates. Her client disagrees with Mary's assumptions regarding inflation and other economic variables used in the retirement needs analysis calculation. What should Mary do next? A)If Mary and her client are unable to agree on the assumptions used for the retirement capital needs analysis, Mary should limit the scope of the engagement and exclude retirement capital needs analysis from her recommendations. B)Mary should use the assumptions that result in the most conservative recommendations for retirement funding. C)The CFP Board's Standards of Professional Conduct require Mary to disengage from the client. D)Mary should provide her client with multiple projections, consistent with all varying assumptions.

The correct answer is A. According to Practice Standard 300-1: Analyzing and Evaluating the Client's Information, the practitioner will utilize client-specified, mutually agreed upon, and/or other reasonable assumptions. If the practitioner and client are unable to agree upon assumptions regarding the capital needs analysis for retirement, it may be appropriate to amend the scope of the engagement and/or to obtain additional information. Answer B is incorrect because Practice Standard 300-1 requires assumptions to be client-specified and mutually agreed upon, not the assumptions that result in the most conservative recommendations. Answer C is incorrect because Mary is not required to disengage, but instead limit the scope of the engagement. Answer D is incorrect because the client disagrees with her assumptions, so either limit the scope of the engagement or use client-specified assumptions.

On December 31st Lisa purchased a home with a 20-year mortgage for $150,000 and an 8% annually compounding interest rate. What is Lisa's principal reduction for the first year? A)$1,814 B)$3,171 C)$2,507 D)$3,912

The correct answer is B. N = 20 x 12 = 240 i = 8/12 = .6667 PV = 150,000 PMT = ? FV = 0 PMT = 1,254.66 12C 12f AMORT X/Y Key On the 10BII, enter 1 input, 12 Orange shift AMORT =

Jack Jones, age 40, earns $100,000 per year and wants to establish a defined contribution plan to encourage employees to stay with his firm. He employs four people whose combined salaries are $60,000 and who range in age from 23 to 30. The average period of employment is 3.5 years. Which vesting schedule is best suited for Jack's plan? A)3-year cliff vesting. B)3-7 year graded vesting. C)5-year cliff vesting. D)2-6 year graded vesting.

The plan is a DC plan so the only vesting schedules allowed are Answers "A" and "D". The only graded vesting schedule is "D". Graded vesting encourages and rewards employee longevity. The graded vesting is probably better since all of his employees would already be 100% vested if they used the 3 year cliff.


Related study sets

Milk Production & Breast Anatomy (139-146)

View Set

BUL3130 Ch 23: Corporate Powers and Management

View Set